Inscription / Connexion Nouveau Sujet
Niveau terminale
Partager :

Calcul de limites ~Fonction Arctan~

Posté par
Awram2000
29-10-20 à 16:45

Bonjour, j'espère que vous allez bien !
Alors aujourd'hui je vous propose un calcul de limites que j'ai discuté avec mes amies, on a bloqué un peu sur le calcul et toutes nos méthodes ont donné un seul résultat  : F.I.

L'énoncé :
Calculer la limite :
Lim x—>+∞ ∛x[arctan(√2+∛x)-arctan(⁶√x)]
Alors on a utilisé la relation /2 =arctan(x) + arctan(1/x). Puis poser t = arctan(U(x)) pour que ça donne tant/t = 1 (inverse d'une limite usuelle) et ça n'a quand même pas marché.
Auriez vous d'autres pistes de réflexion si possible, merci !

Posté par
Awram2000
re : Calcul de limites ~Fonction Arctan~ 29-10-20 à 20:01

Devrais-je éclaircir le raisonnement qu'on a fait ?

Posté par
LeHibou
re : Calcul de limites ~Fonction Arctan~ 29-10-20 à 21:20

Bonjour,

Il me semble qu'une première simplification serait de poser \sqrt[6]{x} = y, d'où \sqrt[3]{x} = y^{2}
Quand x -> +, y -> +
Je ne dis pas que ça résout le problème, mais ça gagne en visibilité.
Ensuite, il y a une jolie formule qui dit :
arctan(a) + arctan(b) = arctan(\frac{a+b}{1-ab}) si ab<1
arctan(a) + arctan(b) = arctan(\frac{a+b}{1-ab}) + si ab>1
Sachant enfin que la fonction arctangente est impaire, donc qu'on peut écrire :
-arctan(y) = + arctan(-y)
On peut peut-être ramener la formule de l'énoncé à une des formules d'addition des arctan, ce qui permettrait d'y voir nettement plus clair, et particulier de lever l'indétermination +-

Posté par
Awram2000
re : Calcul de limites ~Fonction Arctan~ 30-10-20 à 00:27

Bonjour, 😼

Merci pour la piste, par contre, j'ai eu quelques difficultés à l'appliquer :

1/ Elle n'a pas été démontrée en classe, et le sujet de l'exo était dans DS précédent, je présume alors qu'on devrait nous même la démontrer, ou qu'elle ne soit pas vraiment utile pour cet exo, sinon, aurais tu un lien de la démonstration ? Ou la fiche avec cette formule ? Merci.

2/ J'ai appliquée la formule, résultat : F.I.

Je ne sais pas si mon raisonnement/ mes résultats sont correctes, donc je voudrais les revoir avec toi, si possible, (merci d'avance).

P.S : les numéros 1, 2 et 3 correspondent aux expressions présentes dans l'image, j'ai essayé d'écrire avec latex sur le forum et ça s'est avéré trop difficile, j'en suis désolée.

J'ai dit que :

(1)

Ce qui va nous donner l'expression :

(2)

Quand j'ai fini par « factoriser » l'expression pour enlever le y, ça m'a donné :

(3)

Ce qui nous donne (en limites) :

Arctan (0) = 0

Et 0 * = F.I

Calcul de limites ~Fonction Arctan~

Posté par
Awram2000
re : Calcul de limites ~Fonction Arctan~ 30-10-20 à 00:30

Je n'ai pas précisé pour la première mais la justification est que la racine est toujours positive, le carré est toujours positif, donc leur produit est supérieur à 0, donc le - de l'expression est négatif et donc inférieur strictement à 1

Posté par
Foxdevil
re : Calcul de limites ~Fonction Arctan~ 30-10-20 à 01:58

Bonsoir,

La formule de LeHibou permet d'aboutir, mais n'est pas nécessaire.
Par contre pour l'appliquer, il faudrait juste que tu vérifies tes calculs. Au niveau des racines, c'est pas bon. Et tu as inversé y et y^2.

Sinon avec la formule que tu mentionnes, on s'en sort.
La formule donne  y \times ( \frac{\arctan (\frac{1}{y})}{\frac{1}{y}} - \frac{y}{\sqrt2 + y^2} \frac{\arctan (\frac{1}{\sqrt2 + y^2})}{\frac{1}{\sqrt2 + y^2}} ) (après avoir fait apparaître le terme qui est dans le deuxième arc. Et on reconnaît les taux d'accroissement...

(Il faudra aussi la même astuce pour aboutir à partir de l'autre formule )

Posté par
Foxdevil
re : Calcul de limites ~Fonction Arctan~ 30-10-20 à 02:05

(Ou alors j'ai mal compris l'expression. Peut-être que la racine est dans tout le arctan du coup...)

Posté par
Awram2000
re : Calcul de limites ~Fonction Arctan~ 30-10-20 à 08:33

Bonjour,

Merci pour tes remarques et ton aide.

Par contre, je n'ai pas totalement compris l'utilisation du taux d'accroissement dans cet exo, qui je le pense, se définit par l'expression  :

Posté par
Awram2000
re : Calcul de limites ~Fonction Arctan~ 30-10-20 à 08:34

Désolée, voici l?image :

* Modération > Image effacée. Merci d'utiliser les outils mis à ta disposition pour écrire les formules mathématiques.
Celles de 00h27 sont tolérées alors que les 2 premières pouvaient s'écrire... *

Posté par
Awram2000
re : Calcul de limites ~Fonction Arctan~ 30-10-20 à 08:44

+ je ne sais si par les « racines » tu demandais si elle englobait toute l'expression 2 +y2. Dans le doute, je te réponds par l'affirmative.

Posté par
Foxdevil
re : Calcul de limites ~Fonction Arctan~ 30-10-20 à 08:53

Oui c'était bien ce que je voulais dire. Ok.

Alors pour le moment la formule de LeHibou permet de conclure.

Je vais voir s'il y a une manière de faire sans...

Posté par
Awram2000
re : Calcul de limites ~Fonction Arctan~ 30-10-20 à 09:41

Merci beaucoup 😊

De mon côté je vais refaire avec la méthode de LeHibou (en essayant de ne pas me tromper cette fois )

Posté par
Foxdevil
re : Calcul de limites ~Fonction Arctan~ 30-10-20 à 10:00

Alors on peut s'en tirer. Mais ça me paraît quand même plutôt ardu...

Avec plusieurs questions intermédiaires. Quasiment un dm quoi

Si t'es open, je mets les questions subsidiaire pour arriver au résultat.

En attendant, je vois si je trouve plus simple...

Posté par
Foxdevil
re : Calcul de limites ~Fonction Arctan~ 30-10-20 à 10:00

Awram2000 @ 30-10-2020 à 09:41

Merci beaucoup 😊

De mon côté je vais refaire avec la méthode de LeHibou (en essayant de ne pas me tromper cette fois )


Bon courage!

Posté par
Awram2000
re : Calcul de limites ~Fonction Arctan~ 30-10-20 à 10:39

Pour le coup, je ne sais plus quoi faire maintenant 😭 La question est tirée d'un DS (de 2h) de l'année dernière, et elle est sur 1 point voir moins, donc soit :

1/ L'énoncé est faux, ce qui est « possible » puisque j'ai su que la limite tendait vers + l'infini par le biais de l'une de mes amies.

2/ C'est une limite avec une astuce plus au moins simple qu'on a manqué. (Donc pas avec la méthode de LeHibou puisqu'elle n'a pas été étudiée en classe)

P.S : Oui je suis intéressée par ces questions subsidiaires  

Posté par
Foxdevil
re : Calcul de limites ~Fonction Arctan~ 30-10-20 à 11:00

Ok, je te poste ça un peu plus tard

Te serait-il possible de nous recopier l'exercice entier où se trouve la question s'il te plaît ? Il est probable que le contexte permette facilement le calcul...

Sinon ton amie s'est trompée. La limite est 0. Tu peux tracer la fonction pour t'en convaincre

Posté par
Awram2000
re : Calcul de limites ~Fonction Arctan~ 30-10-20 à 11:07

L'exercice demande juste de calculer 4 limites (indépendamment) donc je ne sais pas si ça servirait à grand chose

Pour la limite, tu parlais du résultat ? Puisque je faisais allusion à l'énoncé (lim x—> + l'infini)

Posté par
Foxdevil
re : Calcul de limites ~Fonction Arctan~ 30-10-20 à 11:29

Awram2000 @ 30-10-2020 à 11:07

L'exercice demande juste de calculer 4 limites (indépendamment) donc je ne sais pas si ça servirait à grand chose

Pour la limite, tu parlais du résultat ? Puisque je faisais allusion à l'énoncé (lim x—> + l'infini)
Ok.

Oui je parlais du résultat. Pardon, je n'avais pas bien compris ta remarque

Posté par
Awram2000
re : Calcul de limites ~Fonction Arctan~ 30-10-20 à 11:46

Ce n'est pas grave, d'ailleurs, merci encore pour ton aide, je t'en suis reconnaissante

Posté par
Foxdevil
re : Calcul de limites ~Fonction Arctan~ 30-10-20 à 12:18

Donc on peut y arriver autrement qu'avec la formule suggérée par LeHibou. Mais on est au même niveau de difficulté...

Tout d'abord, on montre que \frac{\tan u - u}{u^3} tend vers 1/3  en 0.

Tu as 3 demos différentes de ce résultat ici Limites.
Elles sont données dans les grandes lignes. N'hésite pas à poser des questions si tu rencontres un problème dans les justifications (qui du coup ne sont pas minutieusement développées).

Une fois cette limite obtenue, on change l'expression dont on veut la limite. On a:

y^2 ( \arctan(\frac{1}{y}) - \arctan(\frac{1}{\sqrt{2+y^2}}))
= \frac{1}{y} ( \frac{\arctan(\frac{1}{y}) - \arctan(\frac{1}{\sqrt{2+y^2}})}{\frac{1}{y^3}} )
= \frac{1}{y} ( \frac{\arctan(\frac{1}{y}) - \frac{1}{y} - ( \arctan(\frac{1}{\sqrt{2+y^2}})  - \frac{1}{\sqrt{2+y^2}}) +\frac{1}{y} - \frac{1}{\sqrt{2+y^2}} }{\frac{1}{y^3}} )

Ce qui est tout à droite se traite en mettant au même dénominateur puis multipliant en haut et en bas par la quantité conjuguée.

Chaque arctan se traite en prenant u=\arctan(\frac{1}{y}) et  u' = \arctan(\frac{1}{\sqrt{2+y^2}}); puis en faisant apparaître \tan^3(u) (resp u') dans le quotient. Un taux d'accroissement devrait apparaître en plus de la limite plus haut...

😪😪

Posté par
Awram2000
re : Calcul de limites ~Fonction Arctan~ 30-10-20 à 12:25

Bon...

Je vais privilégier quelques exos corrigés avant de passer à cette limite, puis je vais essayer de la résoudre plus tard dans le soir.

Posté par
Foxdevil
re : Calcul de limites ~Fonction Arctan~ 30-10-20 à 12:32

De rien. Avec plaisir

Par contre, petite remarque.

Citation :
L'énoncé :
Calculer la limite :
Lim x—>+∞ ∛x[arctan(√2+∛x)-arctan(⁶√x)]
Essaie de faire attention à l'écriture. La parenthèse dans la racine du premier arctan change radicalement la signification et difficulté de l'exercice. C'est un peu rageant de passer du temps sur la mauvaise question "pour rien" 😅

Posté par
Foxdevil
re : Calcul de limites ~Fonction Arctan~ 30-10-20 à 12:33

Awram2000 @ 30-10-2020 à 12:25

Bon...

Je vais privilégier quelques exos corrigés avant de passer à cette limite, puis je vais essayer de la résoudre plus tard dans le soir.

Je te comprends parfaitement! Tu as bien raison! 😂😂😉



Vous devez être membre accéder à ce service...

Pas encore inscrit ?

1 compte par personne, multi-compte interdit !

Ou identifiez-vous :


Rester sur la page

Inscription gratuite

Fiches en rapport

parmi 1675 fiches de maths

Désolé, votre version d'Internet Explorer est plus que périmée ! Merci de le mettre à jour ou de télécharger Firefox ou Google Chrome pour utiliser le site. Votre ordinateur vous remerciera !